上一主题:Equity Investments【Reading 50】Sample
下一主题:Equity Investments【Reading 47】Sample
返回列表 发帖
A stock is said to be undervalued if its market price is:
A)
greater than its intrinsic value.
B)
less than its intrinsic value.
C)
less than its book value.



A security with a market price less than its intrinsic value is undervalued.

TOP

Which of the following is NOT an assumption behind efficient capital markets?
A)
Market participants correctly adjust prices to reflect new information.
B)
Return expectations implicitly include risk.
C)
New information occurs randomly, and the timing of announcements is independent of one another.



The set of assumptions that imply an efficient capital market includes:
  • There exists a large number of profit-maximizing market participants.
  • New information occurs randomly.
  • Market participants adjust their price expectations rapidly (but not necessarily correctly).
  • Return expectations implicitly include risk.

TOP

Which of the following statements on the forms of the efficient market hypothesis (EMH) is least accurate?
A)
The semi-strong form EMH addresses market and non-market public information.
B)
The strong-form EMH assumes perfect markets.
C)
The weak-form EMH states that stock prices reflect current public market information and expectations.



The weak-form EMH assumes the price of a security reflects all currently available historical information. Thus, the past price and volume of trading has no relationship with the future, hence technical analysis is not useful in achieving superior returns.
The other statements are true. The strong-form EMH states that stock prices reflect all types of information: market, non-public market, and private. No group has monopolistic access to relevant information; thus no group can achieve excess returns. For these assumptions to hold, the strong-form assumes perfect markets – information is free and available to all.

TOP

The statement, "Stock prices fully reflect all information from public and private sources," can be attributed to which form of the efficient market hypothesis (EMH)?
A)
Semistrong-form EMH.
B)
Weak-form EMH.
C)
Strong-form EMH.



This is the definition of the strong-form EMH. Private sources include insider information, such as persons holding monopolistic access to information relevant to the formation of prices.

TOP

The strong-form efficient market hypothesis (EMH) asserts that stock prices fully reflect which of the following types of information?
A)
Public and private.
B)
Public, private, and future.
C)
Market.



The strong-form EMH assumes that stock prices fully reflect all information from public and private sources.

TOP

The semi-strong form of efficient market hypothesis (EMH) asserts that:
A)
all public information is already reflected in security prices.
B)
both public and private information is already incorporated into security prices.
C)
past and future prices exhibit little or no relationship to another.



Semi-strong EMH states that publicly available information cannot be used to consistently beat the market performance.

TOP

Which of the following statements about market efficiency is least accurate?
A)
The weak-form EMH suggests that fundamental analysis will not provide excess returns while the semi-strong form suggests that technical analysis cannot achieve excess returns.
B)
The strong-form EMH assumes cost free availability of all information, both public and private.
C)
The semi-strong form EMH addresses market and non-market public information.


The weak-form EMH suggests that technical analysis will not provide excess returns while the semi-strong form suggests that fundamental analysis cannot achieve excess returns. The weak-form EMH assumes the price of a security reflects all currently available historical information. Thus, the past price and volume of trading has no relationship with the future, hence technical analysis is not useful in achieving superior returns.
The other choices are correct. The strong-form EMH states that stock prices reflect all types of information: market, non-public market, and private. No group has monopolistic access to relevant information; thus no group can achieve excess returns. For these assumptions to hold, the strong-form assumes perfect markets – information is free and available to all.

TOP

Which of the following is least likely an assumption behind the semistrong-form of the efficient market hypothesis (EMH)?
A)
The timing of news announcements are independent of each other.
B)
All information is cost-free and available to everyone at the same time.
C)
A large number of profit-maximizing participants.



The strong-form EMH assumes all information, both public and private, is cost-free and available to all investors at the same time.

TOP

Which of the following forms of the EMH assumes that no group of investors has monopolistic access to relevant information?
A)
Strong-form.
B)
Both weak and semistrong form.
C)
Weak-form.



The strong-form EMH assumes that stock prices fully reflect all information from public and private sources. In addition, no group of investors has monopolistic access to information relevant to the formation of prices.

TOP

The semi-strong form of the efficient market hypothesis (EMH) asserts that stock prices:
A)
fully reflect all historical price information.
B)
fully reflect all relevant information including insider information.
C)
fully reflect all publicly available information.



The semi-strong form of the EMH asserts that security prices fully reflect all publicly available information. This would include all historical information. The weak form relates to historical information only. The strong form relates to public and private information.

TOP

返回列表
上一主题:Equity Investments【Reading 50】Sample
下一主题:Equity Investments【Reading 47】Sample